Difference between revisions of "2005 AMC 10B Problems/Problem 5"

(Problem)
(Solution)
Line 6: Line 6:
  
 
== Solution ==
 
== Solution ==
{{solution}}
+
Since she will use <math>\frac{1} {5}</math> of her money to buy <math>\frac{1} {3}</math> of the CD's, she will use <math>\frac{3} {5}</math> of her money to buy all the CD's. Thus, she will have <math>1-\frac{3} {5}=\frac{2} {5}</math> of her money left. Hence, the answer is <math>\boxed{C}</math>
 +
 
 
== See Also ==
 
== See Also ==
 
*[[2005 AMC 10B Problems/Problem 4 | Previous problem]]
 
*[[2005 AMC 10B Problems/Problem 4 | Previous problem]]
 
*[[2005 AMC 10B Problems/Problem 6 | Next problem]]
 
*[[2005 AMC 10B Problems/Problem 6 | Next problem]]
 
*[[2005 AMC 10B Problems]]
 
*[[2005 AMC 10B Problems]]

Revision as of 12:20, 27 August 2008

This is an empty template page which needs to be filled. You can help us out by finding the needed content and editing it in. Thanks.

Problem

Brianna is using part of the money she earned on her weekend job to buy several equally priced CDs. She used one fifth of the money to buy one third of the CD's. What fraction of the money will she have left once she buys the CD's?

$\mathrm{(A)}\ {{{\frac{1} {5}}}} \qquad \mathrm{(B)}\ {{{\frac{1} {3}}}} \qquad \mathrm{(C)}\ {{{\frac{2} {5}}}} \qquad \mathrm{(D)}\ {{{\frac{2} {3}}} \qquad \mathrm{(E)}\ {{{\frac{4} {5}}}}$ (Error compiling LaTeX. Unknown error_msg)

Solution

Since she will use $\frac{1} {5}$ of her money to buy $\frac{1} {3}$ of the CD's, she will use $\frac{3} {5}$ of her money to buy all the CD's. Thus, she will have $1-\frac{3} {5}=\frac{2} {5}$ of her money left. Hence, the answer is $\boxed{C}$

See Also